Let f be a continuous function on the closed interval [1,5], where f(1)=1 and f(5)=−3.Which of the following is guaranteed by the Intermediate Value Theorem?Choose 1 answer:(A) f(c)=−2 for at least one c between 1 and 5(B) f(c)=−2 for at least one c between −3 and 1(C) f(c)=2 for at least one c between 1 and 5(D) f(c)=2 for at least one c between −3 and 1
Q. Let f be a continuous function on the closed interval [1,5], where f(1)=1 and f(5)=−3.Which of the following is guaranteed by the Intermediate Value Theorem?Choose 1 answer:(A) f(c)=−2 for at least one c between 1 and 5(B) f(c)=−2 for at least one c between −3 and 1(C) f(c)=2 for at least one c between 1 and 5(D) f(c)=2 for at least one c between −3 and 1
Apply Theorem to Function: The Intermediate Value Theorem states that if f is a continuous function on a closed interval [a,b] and N is any number between f(a) and f(b), then there exists at least one c in the interval (a,b) such that f(c)=N. We need to apply this theorem to the given function f.
Evaluate Function Endpoints: Since f(1)=1 and f(5)=−3, we know that the function values at the endpoints of the interval [1,5] are 1 and −3, respectively. The Intermediate Value Theorem will apply to any value N that lies between 1 and −3.
Determine Guaranteed Value: Looking at the answer choices, we need to determine which value of N is guaranteed to have a corresponding c in the interval (1,5) such that f(c)=N.
Analyze Choice (A): Choice (A) suggests that f(c)=−2 for some c between 1 and 5. Since −2 is between 1 and −3, the Intermediate Value Theorem guarantees that there is at least one c in the interval (1,5) for which f(c)=−2.
Reject Choice (B): Choice (B) is incorrect because it refers to a c between −3 and 1, which is outside the interval [1,5] we are considering.
Reject Choice (C): Choice (C) suggests that f(c)=2 for some c between 1 and 5. However, since 2 is not between 1 and −3, the Intermediate Value Theorem does not guarantee a c in the interval (1,5) such that f(c)=2.
Reject Choice (D): Choice (D) is incorrect for the same reason as choice (B); it refers to a c between −3 and 1, which is outside the interval [1,5].
More problems from Domain and range of quadratic functions: equations